LSAT and Law School Admissions Forum

Get expert LSAT preparation and law school admissions advice from PowerScore Test Preparation.

 Administrator
PowerScore Staff
  • PowerScore Staff
  • Posts: 8919
  • Joined: Feb 02, 2011
|
#85204
Complete Question Explanation

Strengthen, Except—Cause and Effect. The correct answer choice is (C).

Answer choice (A):

Answer choice (B):

Answer choice (C): This is the correct answer choice.

Answer choice (D):

Answer choice (E):

This explanation is still in progress. Please post any questions below!
 nickp18
  • Posts: 20
  • Joined: May 26, 2020
|
#88095
Hi Powerscore Team!

I got this question wrong the first time, but correct during my blind review. I had trouble identifying the conclusion, so will you please check my reasoning on this?

Since we have a strengthen-except question, we know that the four incorrect answers will each strengthen the argument, and the correct answer will either A) weaken or B) have no effect on the argument.

Sub-conclusion: Quitting smoking is difficult to do.
Main conclusion: (the key to quitting) may be as simple as replacing an unhealthy activity with a healthy one.

Answer A. Supports that exercise prevents weight gain, when weight gain is one of the reasons quitting is difficult.

Answer B. 400 randomly selected patients helps assure the study was credible and not biased by members unrepresentative of the population.

Answer C. Does nothing to improve the quality or strength of the conclusion.

Answer D. Increased endorphins and reduced tension leads to less stress potentially; making the stress aspect of quitting easier.

Answer E. 38% success rate after the one year mark shows that this is a sustainable habit, meaning exercise is beneficial to quitting smoking.


Thank you!

Nick
User avatar
 Poonam Agrawal
PowerScore Staff
  • PowerScore Staff
  • Posts: 71
  • Joined: Apr 23, 2021
|
#88140
Hi Nick!

Your reasoning looks great, and the conclusion you've identified is spot on. The main issue with answer choice (C), as you've noted, is that it doesn't add any support to the argument which is why it is the correct answer here. The rest of the answers either add credibility to the study or help explain the study's findings, which do support the argument.

Awesome job!

Get the most out of your LSAT Prep Plus subscription.

Analyze and track your performance with our Testing and Analytics Package.